Đến nội dung

Hình ảnh

Topic: [LTDH] Mỗi ngày hai bất đẳng thức.


  • Chủ đề bị khóa Chủ đề bị khóa
Chủ đề này có 215 trả lời

#201
tritanngo99

tritanngo99

    Đại úy

  • Điều hành viên THPT
  • 1644 Bài viết

Tiếp theo: 

Bài 97: Cho $x,y,z$ là các số thực dương. Chứng minh rằng:

$\frac{2x^2+xy}{(y+\sqrt{zx}+z)^2}+\frac{2y^2+yz}{(z+\sqrt{xy}+x)^2}+\frac{2z^2+zx}{(x+\sqrt{yz}+y)^2}\ge 1$.

Bài 98: Cho ba số thực dương $a,b,c$ thỏa mãn: $ab+bc+ca=3abc$. Tìm GTNN của biểu thức:

$P=(\frac{a}{b}-\frac{1}{b}+\frac{1}{ab})(\frac{b}{c}-\frac{1}{c}+\frac{1}{bc})(\frac{c}{a}-\frac{1}{a}+\frac{1}{ca})$



#202
MrS

MrS

    Binh nhất

  • Thành viên
  • 29 Bài viết

Tiếp theo: 

Bài 97: Cho $x,y,z$ là các số thực dương. Chứng minh rằng:

$\frac{2x^2+xy}{(y+\sqrt{zx}+z)^2}+\frac{2y^2+yz}{(z+\sqrt{xy}+x)^2}+\frac{2z^2+zx}{(x+\sqrt{yz}+y)^2}\ge 1$.

Bài 97:

Sử dụng Cauchy - Schwarz: $(y+\sqrt{zx}+z)^2\leq (y+z+z)(y+x+z)=(x+y+z)(y+2z)$

Từ đó ta chỉ cần chứng minh: $P=\sum \frac{2x^2+xy}{y+2z}\geq x+y+z$

Theo Cauchy - Schwarz: $P\geq \frac{(2\sum x^2+\sum xy)^2}{\sum (y+2z)(2x^2+xy)}=\frac{(2\sum x^2+\sum xy)^2}{2(xy+yz+zx)(x+y+z)+3(xy^2+yz^2+zx^2+xyz)-3xyz}=M$

Áp dụng bổ đề quen thuộc: $xy^2+yz^2+zx^2+xyz\leq \frac{4(x+y+z)^3}{27}$

Suy ra: $M\geq \frac{(2\sum x^2+\sum xy)^2}{2(xy+yz+zx)(x+y+z)+\frac{4(x+y+z)^3}{9}-3xyz}$

Đưa về dạng $p, q, r$ và chuẩn hóa $p=1$

Ta đi chứng minh: $\frac{(2-3q)^2}{2q+\frac{4}{9}-3r}\geq 1\Leftrightarrow 81q^2-126q+32+27r\geq 0$ (*)

Theo BĐT Schur thì: $r\geq \frac{p(4q-p^2)}{9}=\frac{4q-1}{9}$

Nên $VT(*)\geq (1-3q)(29-27q)\geq 0$ hiển nhiên đúng do $0< q\leq \frac{1}{3}< \frac{29}{27}$

Bài toán được chứng minh. Đẳng thức xảy ra khi  $x=y=z$


Bài viết đã được chỉnh sửa nội dung bởi MrS: 16-10-2016 - 03:57


#203
MrS

MrS

    Binh nhất

  • Thành viên
  • 29 Bài viết

Tiếp theo: 

Bài 98: Cho ba số thực dương $a,b,c$ thỏa mãn: $ab+bc+ca=3abc$. Tìm GTNN của biểu thức:

$P=(\frac{a}{b}-\frac{1}{b}+\frac{1}{ab})(\frac{b}{c}-\frac{1}{c}+\frac{1}{bc})(\frac{c}{a}-\frac{1}{a}+\frac{1}{ca})$

 

Bài 98: Đổi: $(\frac{1}{a}-1,\frac{1}{b}-1,\frac{1}{c}-1)\rightarrow (x,y,z)$

Khi đó $P=(x^2+x+1)(y^2+y+1)(z^2+z+1)$ với $x+y+z=0$

Thế $z=-x-y$ vào P ta sẽ CM: $(x^2+x+1)(y^2+y+1)((x+y)^2-x-y+1)-1\geq 0$ (*)

Đặt $x+y=S; xy =P$ BĐT trên trở thành: $f(P)=(S^2-S+1)P^2+(S-1)(S^2-S+1)P+S^4+S^2\geq 0$

Không mất tổng quá, giả sử $P=xy\geq 0$ lại có $z>-1\Rightarrow S=x+y=-z<1$ và $S^2\geq 4P$

Ta có: $f"(P)=2(S^2-S+1)>0\Rightarrow f'(P)\geq f'(0)=(S-1)(S^2-S+1)<0\Rightarrow f(P)\geq f(\frac{S^2}{4})=\frac{S^2}{16}((S^2+\frac{3S}{2})^2+4(S+1)^2+\frac{11S^2}{4}+8)\geq 0$

(*) được CM. Vậy $MinP=1$ tại $a=b=c=1$


Bài viết đã được chỉnh sửa nội dung bởi MrS: 16-10-2016 - 03:32


#204
tritanngo99

tritanngo99

    Đại úy

  • Điều hành viên THPT
  • 1644 Bài viết

Bài 97 và bài 98: MrS đã cho lời giải đúng.

Tiếp theo: 

Bài 99: Xét các đa thức: $P(x)=x^3-6x^2+mx-n$ với $m,n\in \mathbb{R}$ có $3$ nghiệm thuộc đoạn: $[1;3]$. Tìm đa thức sao cho $m$ đạt GTNN.

Bài 100: Cho $x,y,z$ là độ dài 3 cạnh tam giác có chu vi bằng $3$. Chứng minh rằng:

$\frac{x}{y+z}+\frac{y}{z+x}+\frac{z}{x+y}\le \frac{1}{3}(\frac{1}{x}+\frac{1}{y}+\frac{1}{z})+\frac{3}{xy+yz+zx}$.



#205
tritanngo99

tritanngo99

    Đại úy

  • Điều hành viên THPT
  • 1644 Bài viết

Lời giải bài 99 và bài 100:

Lời giải bài 99: 

Gọi $3$ nghiệm của $P(x)$ là $x_1,x_2,x_3\in[1;3]$. Theo định lí Viet ta có:

$\left\{\begin{matrix} x_1+x_2+x_3=6 \\x_1x_2+x_2x_3+x_3x_1=m  \end{matrix}\right.$.

Sử dụng bổ để quen thuộc: 

Bổ đề: Cho $a,b,c\in [0,2]$ thỏa mãn: $a+b+c=3$. Khi đó: $ab+bc+ca\ge 2$. Dấu $=$ xảy ra tại: $(a,b,c)=(0,1,2)$ và các hoán vị.

Trở lại bài toán: Áp dụng bổ đề với ba số: $x_1-1,x_2-1,x_3-1\in [0;2]$ ta được:

$(x_1-1)(x_2-1)+(x_2-1)(x_3-1)+(x_3-1)(x_1-1)\ge 2\implies x_1x_2+x_2x_3+x_3x_1\ge 2(x_1+x_2+x_3)-1\implies m\ge 11$.

Dấu $=$ xảy ra khi: $(x_1,x_2,x_3)=(1;2;3)$ và các hoán vị.

Với $m=11$. Khi đó: $P(x)=(x-1)(x-2)(x-3)=x^3-6x^2+11x-6$.

Lời giải bài 100: Áp dụng kết quả của bài TST 2006.( Các bạn có thể xem tại đây:http://diendantoanho...nam-tst-2006/).

Đó là: $6(\frac{x}{y+z}+\frac{y}{z+x}+\frac{z}{x+y})\le (x+y+z)(\frac{1}{x}+\frac{1}{y}+\frac{1}{z})$.

Do đó: $\frac{x}{y+z}+\frac{y}{z+x}+\frac{z}{x+y}\le \frac{1}{2}(\frac{1}{x}+\frac{1}{y}+\frac{1}{z})$.

Dễ chứng minh được: $\frac{x}{y+z}+\frac{y}{z+x}+\frac{z}{x+y}+\frac{xy+yz+zx}{x^2+y^2+z^2}\le \frac{5}{2}$.

(Phần chứng minh dành cho bạn đọc).

Từ hai kết quả trên ta có:

$\frac{x}{y+z}+\frac{y}{z+x}+\frac{z}{x+y}=\frac{2}{3}\sum \frac{x}{y+z}+\frac{1}{3}\sum \frac{x}{y+z}\le \frac{1}{3}(\sum \frac{1}{x})+\frac{1}{3}(\frac{5}{2}-\frac{xy+yz+zx}{x^2+y^2+z^2})=\frac{5}{6}+\frac{1}{3}(\frac{1}{x}+\frac{1}{y}+\frac{1}{z})-\frac{xy+yz+zx}{3(x^2+y^2+z^2)}$.

Vậy chỉ cần chứng minh bước cuối cùng là BDT sau: $\frac{3}{xy+yz+zx}+\frac{xy+yz+zx}{x^2+y^2+z^2}\ge \frac{4}{3}$.

(Phần CM dành cho bạn đọc).


Bài viết đã được chỉnh sửa nội dung bởi tritanngo99: 17-10-2016 - 15:50


#206
tritanngo99

tritanngo99

    Đại úy

  • Điều hành viên THPT
  • 1644 Bài viết

Tiếp theo:

Bài 101: Cho $x,y$ là hai số thực dương thỏa mãn: $x^3+y^3=1$. Tìm GTNN của biểu thức:

$A=\frac{x^2+y^2}{(1-x)(1-y)}$.

Bài 102: Cho $a,b,c$ là độ dài ba cạnh của một tam giác. Chứng minh:

$\sqrt{\frac{a}{b+c-a}}+\sqrt{\frac{b}{a+c-b}}+\sqrt{\frac{c}{b+a-c}}\ge 3$.



#207
loolo

loolo

    Trung sĩ

  • Thành viên
  • 198 Bài viết

Lời giải bài 102:

Bổ đề quen thuộc với a,b,c là 3 cạnh của 1 tam giác:

                                   $(b+c-a)(a+c-b)(a+b-c)\leq abc$

Áp dụng bổ đề ta được: $\sqrt{\frac{a}{b+c-a}}+\sqrt{\frac{b}{a+c-b}}+\sqrt{\frac{c}{a+b-c}}\geq 3\sqrt[6]{\frac{abc}{(a+b-c)(b+c-a)(a+c-b)}}\geq 3\sqrt[6]{\frac{abc}{abc}}=3$ (đpcm)

Dấu " = " tại a=b=c.


Bài viết đã được chỉnh sửa nội dung bởi loolo: 17-10-2016 - 16:34

 


#208
tritanngo99

tritanngo99

    Đại úy

  • Điều hành viên THPT
  • 1644 Bài viết

Dưới đây là lời giải bài 101 và cách khác bài 102:

Lời giải bài 101:

Đặt: $(s;p)\to (x+y;xy)$.

Ta có: $x^3+y^3=1\iff s^3-3ps=1\implies p=\frac{s^3-1}{3s}\to s>1$.

Mặt khác: $(x+y)^3\le 4(x^3+y^3)=4\to s\le \sqrt[3]{4}$.

Ta có: $A=\frac{x^2+y^2}{(1-x)(1-y)}=\frac{s^2-2p}{p+1-s}=\frac{s^3+2}{(s-1)^3}$.

Xét hàm số: $f(s)=\frac{s^3+2}{(s-1)^3}\forall s\in (1;\sqrt[3]{4}]$. Ta có: $f'(s)=\frac{-3(s^2+2)}{(s-1)^4}<0$ nên hàm $f(s)$ nghịch biến trên $(1;\sqrt[3]{4}]$.

Suy ra: $f(s)\ge f(\sqrt[3]{4})=\frac{6}{(\sqrt[3]{4}-1)^3}$.

Vậy GTNN của $A$ là: $\frac{6}{(\sqrt[3]{4}-1)^3}$, đạt được khi $x=y=\frac{1}{\sqrt[3]{2}}$.

Lời giải bài 102:

Áp dụng $AM-GM$ ta có: $2\sqrt{\frac{b+c-a}{a}}\le \frac{b+c-a}{a}+1=\frac{b+c}{a}\implies \sum \sqrt{\frac{a}{b+c-a}}\ge 2\sum \frac{a}{b+c}\ge 3(\text{ Nesbit})\implies Q.E.D$.


Bài viết đã được chỉnh sửa nội dung bởi tritanngo99: 19-10-2016 - 15:52


#209
tritanngo99

tritanngo99

    Đại úy

  • Điều hành viên THPT
  • 1644 Bài viết

Tiếp theo:

Bài 103: Cho $a,b,c$ là các số thực dương thỏa mãn: $a^2+b^2+c^2=5(a+b+c)-2ab$. Tìm GTNN của biểu thức:

$Q=a+b+c+48(\frac{\sqrt{3}}{\sqrt{a+10}}+\frac{1}{\sqrt[3]{b+c}})$.

Bài 104: Cho các số thực dương $a,b,c$ thỏa mãn điều kiện sau:

$a^2b^2c^2+(1+a)(1+b)(1+c)\ge a+b+c+ab+bc+ca+3$.

Tìm GTNN của $S=\sum \frac{a^3}{(b+2c)(2c+3a)}$



#210
tritanngo99

tritanngo99

    Đại úy

  • Điều hành viên THPT
  • 1644 Bài viết

Dưới đây là lời giải bài 103 và bài 104:

Lời giải bài 103:

Ta có: $5(a+b+c)=(a+b)^2+c^2\le \frac{1}{2}(a+b+c)^2\implies 0<a+b+c\le 10$.

Ta có: $2\sqrt{\frac{a+10}{3}}\le \frac{1}{6}(a+22);3\sqrt[3]{b+c}\le \frac{1}{4}(c+b+16)$.

$Q=a+b+c+48(\frac{1}{\sqrt{\frac{a+10}{3}}}+\frac{1}{\sqrt[3]{b+c}})\ge a+b+c+48(\frac{12}{a+22}+\frac{12}{b+c+16})$

$\ge a+b+c+576(\frac{4}{a+b+c+38})=a+b+c+\frac{2304}{a+b+c+38}$.

Đặt $f(t)=t+\frac{2304}{t+38},\forall t\in (0;10]$.

Khảo sát $f(t)\implies f(t)\ge f(10)=58$.

Suy ra $Q_{min}=58$. Khi $(a;b;c)=(2;3;5)$.

Lời giải bài 104:

Từ giả thiết: $a^2b^2c^2+(1+a)(1+b)(1+c)\ge a+b+c+ab+bc+ca+3\iff (abc)^2+abc-2\ge 0\iff abc\ge 1$.

Áp dụng $AM-GM$, ta có:

$\frac{a^3}{(b+2c)(2c+3a)}+\frac{b+2c}{45}+\frac{2c+3a}{75}\ge \frac{a}{5}$.

Tương tự: $S+\frac{2(a+b+c)}{15}\ge \frac{a+b+c}{5}\implies S\ge \frac{1}{15}(a+b+c)\ge \frac{1}{4}$.

Vậy $S_{min}=\frac{1}{5}$. Dấu $=$ xảy ra tại: $a=b=c=1$.



#211
tritanngo99

tritanngo99

    Đại úy

  • Điều hành viên THPT
  • 1644 Bài viết

Tiếp theo:

Bài 105: Cho ba số thực dương $a,b,c$ thỏa mãn: $a+b+c=3$. Tìm GTNN của biểu thức: $P=\sum \frac{b\sqrt{b}}{\sqrt{2a+b+c}}$.

Bài 106: Cho $x,y,z>0$ và $xyz=8$. Tìm GTLN của biểu thức: Q=$\sum \frac{1}{2x+y+6}$



#212
MrS

MrS

    Binh nhất

  • Thành viên
  • 29 Bài viết

Tiếp theo:

Bài 105: Cho ba số thực dương $a,b,c$ thỏa mãn: $a+b+c=3$. Tìm GTNN của biểu thức: $P=\sum \frac{b\sqrt{b}}{\sqrt{2a+b+c}}$.

$\sum \frac{b\sqrt{b}}{\sqrt{2a+b+c}}= \sum \frac{2b^2}{\sqrt{4b(2a+b+c)}}\geq \sum \frac{4b^2}{2a+5b+c}\geq \frac{4(a +b+c)^2}{8(a+b+c)}=\frac{3}{2}$



#213
tritanngo99

tritanngo99

    Đại úy

  • Điều hành viên THPT
  • 1644 Bài viết

Dưới đây là lời giải khác bài 105 và bài 106:

Lời giải bài 105:

Từ giả thiết ta có: $P=\sum \frac{b\sqrt{b}}{\sqrt{a+3}}$.

Áp dụng $AM-GM$ ta có:

$\frac{b\sqrt{b}}{2\sqrt{a+3}}+\frac{b\sqrt{b}}{2\sqrt{a+3}}+\frac{a+3}{16}\ge \frac{3b}{4}$.

Tương tự suy ra:  $\sum \frac{b\sqrt{b}}{\sqrt{a+3}}+\frac{a+b+c+9}{16}\ge \frac{3}{4}(a+b+c)\implies P\ge \frac{3}{2}$.

Lời giải bài 106:

Đặt: $(x;y;z)\to (2a;2b;2c)\implies abc=1$.

$P=\frac{1}{2}\sum \frac{1}{2a+b+3}$.

Áp dụng BDT quen thuộc: $\frac{1}{x+y}\le \frac{1}{4}(\frac{1}{x}+\frac{1}{y})$.

Ta có: $P=\frac{1}{2}\sum \frac{1}{2a+b+3}=\frac{1}{2}\sum \frac{1}{(a+b+1)+(a+2)}\le \frac{1}{8}(\sum \frac{1}{a+b+1}+\sum \frac{1}{a+2})$.

Bây giờ ta sẽ đi chứng minh: $\sum \frac{1}{a+b+1}+\sum \frac{1}{a+2}\le 2$.

Ta sẽ tách thành $2$ phần để chứng minh: $\sum \frac{1}{a+b+1}\le 1;\sum \frac{1}{a+2}\le 1$.

Hai BDT trên dễ dàng chứng minh bằng biến đổi tương đương( Phần CM dành cho bạn đọc).

Từ đây ta tìm được: $Max(P)=\frac{1}{4}$. Dấu $=$ xảy ra tại $(x;y;z)=(2;2;2)$.

 



#214
tritanngo99

tritanngo99

    Đại úy

  • Điều hành viên THPT
  • 1644 Bài viết

Tiếp theo:

Bài 107: Cho các số thực $a,b,c\in [0;1]$ thỏa mãn: $a+b+c=2$. Tìm GTLN của biểu thức: $P=\sum \sqrt{a^2-4a+5}$.

Bài 108: Cho $a,b,c>0$.  Tìm GTNN của biểu thức: $P=\sum \frac{a+2b}{b+\sqrt{ab}}$.



#215
hanguyen445

hanguyen445

    Thượng sĩ

  • Thành viên
  • 240 Bài viết

Tiếp theo:

Bài 107: Cho các số thực $a,b,c\in [0;1]$ thỏa mãn: $a+b+c=2$. Tìm GTLN của biểu thức: $P=\sum \sqrt{a^2-4a+5}$.

Bài 108: Cho $a,b,c>0$.  Tìm GTNN của biểu thức: $P=\sum \frac{a+2b}{b+\sqrt{ab}}$.

Đặt $x=\sqrt{a};y=\sqrt{b};z=\sqrt{c}$. 

Hình gửi kèm

  • Capture67.JPG

Đề thi chọn đội tuyển  HSG:

http://diendantoanho...date-2016-2017/

Topic thảo luận bài toán thầy Hùng:

http://diendantoanho...topicfilter=all

Blog Thầy Trần Quang Hùng

http://analgeomatica.blogspot.com/

Hình học: Nguyễn Văn Linh

https://nguyenvanlin...ss.com/2016/09/

Toán học tuổi trẻ:

http://www.luyenthit...chi-thtt-online

Mathlink:http://artofproblemsolving.com

BẤT ĐẲNG THỨC:

http://diendantoanho...-đẳng-thức-vmf/

http://diendantoanho...i-toán-quốc-tế/

 


#216
tritanngo99

tritanngo99

    Đại úy

  • Điều hành viên THPT
  • 1644 Bài viết

Dưới đây là lời giải bài 107 và cách khác bài 108:

Lời giải bài 107:

Đặt $f(x)=\sqrt{x^2-4x+5}$.

Áp dụng phương pháp tiếp tuyến kết hợp dấu $=$ xảy ra ta đánh giá được:

$f(x)\le (\sqrt{2}-\sqrt{5})x+\sqrt{5}$.

Phần còn lại xin dành cho bạn đọc.

Lời giải bài 108: 

Ta có: $\frac{x^2+2}{x+1}\ge \frac{1}{4}x+\frac{5}{4}\iff \frac{3(x-1)^2}{4(x+1)}\ge 0(TRUE)$.

$\implies \frac{a+2b}{b+\sqrt{ab}}=\frac{(\sqrt{\frac{a}{b}})^2+2}{\sqrt{\frac{a}{b}}+1}\ge \frac{1}{4}\sqrt{\frac{a}{b}}+\frac{5}{4}\forall a,b>0$.

Tương tự rồi cộng lại ta được: $P\ge \frac{1}{4}(\sum \sqrt{\frac{a}{b}})+\frac{15}{4}\ge \frac{3}{4}+\frac{15}{4}=\frac{9}{2}\forall a,b,c>0.$

Ps: TOPIC: [LTDH] Mỗi ngày hai bất đăng thức xin tạm hoãn một thời gian vì để dành thời gian cho ôn thi đại học. Mong mọi người thông cảm và TOPIC sẽ được mở lại vào mùa hè năm sau...






0 người đang xem chủ đề

0 thành viên, 0 khách, 0 thành viên ẩn danh